Fiche de mathématiques
> >

Filtres et limites

Partager :

Le présent cours est repris du travail de Bourbaki.
Il s'agit d'un résumé adapté et facilement lisible du monumental ouvrage : "Éléments de mathématique - Topologie Générale".

§1 - Les filtres

1. Définition d'un filtre


Définition 1
On appelle filtre sur un ensemble X, la donnée d'un sous-ensemble \mathcal F de parties de X vérifiant les conditions suivantes :
\bullet(F_1) Toute partie de X contenant un ensemble de \mathcal F appartient à \mathcal F
\bullet(F_2) Toute intersection finie d'ensembles de \mathcal F appartient à \mathcal F
\bullet(F_3) La partie vide de X n'appartient pas à \mathcal F


Des deux dernières propriétés, on déduit que toute intersection finie d'ensembles de \mathcal F est non vide. Mais rien n'interdit qu'une intersection infinie d'ensembles de \mathcal F soit vide.
L'axiome (F_2) est encore équivalent aux deux suivants :
\bullet(F_2_a) Toute intersection de deux ensembles de \mathcal F appartient à \mathcal F
\bullet(F_2_b)~X \in \mathcal F
Les axiomes (F_2_b) et (F_3) montrent qu'il n'y a pas de filtre sur l'ensemble vide.
Notons que toute réunion non vide d'ensembles de \mathcal F est encore un ensemble de \mathcal F. En effet, la dite réunion contient un ensemble de \mathcal F et il suffit d'appliquer (F_1)

Exemples de filtres:
1) Si X est non vide, alors \mathcal F = \lbrace X \rbrace est un filtre sur X.
D'une manière générale, toutes les parties de X contenant une partie non vide A de X est un filtre sur X.
2) Sur un espace topologique X, l'ensemble des voisinages d'une partie non vide de X (et en particulier, d'un point) est un filtre sur X.
3) Si X est un ensemble infini, l'ensemble des complémentaires des parties finies de X est un filtre sur X. Le filtre des complémentaires des parties finies de \N est appelé le filtre de Fréchet.

2. Comparaison de filtres


On rappelle que si X est un ensemble, l'ensemble \mathcal{P}(\mathcal P(X)) est ordonné par la relation d'inclusion suivante :

\boxed{\large \forall \mathcal A, \mathcal B \in \mathcal P(\mathcal P(X)), \mathcal A \leq \mathcal B \Leftrightarrow \mathcal A \subset \mathcal B}

L'ensemble des filtres sur X est une partie de \mathcal P(\mathcal P(X)) et peut donc être ordonné par la relation ci-dessus décrite. Mais ce n'est pas un ensemble totalement ordonné, d'où :
Définition 2
Étant donnés deux filtres \mathcal F et \mathcal F' sur un ensemble X, on dit que \mathcal F' est plus fin que \mathcal F si \mathcal F \subset \mathcal F'
De tels filtres sont dits comparables.
Si de plus \mathcal F \neq \mathcal F' alors \mathcal F' est dit strictement plus fin que \mathcal F

Dans l'ensemble des filtres sur X, l'élément \lbrace X \rbrace est le plus petit.
Si X possède a plus d'un élément, l'ensemble ordonné des filtres sur X n'a pas de plus grand élément.

Soit (\mathcal F_i)_{i\in I} une famille non vide quelconque de filtres sur un ensemble X.
L'ensemble \displaystyle \mathcal F = \bigcap_{i\in I}\mathcal F_i vérifie les axiomes (F_1),(F_2) et (F_3) et est donc un filtre sur X, appelé filtre intersection de la famille (\mathcal F_i)_{i\in I} et qui est la borne inférieure de l'ensemble des \mathcal F_i dans l'ensemble ordonné des filtres sur X.

Etant donné un ensemble \mathcal S de parties d'un ensemble X, cherchons s'il existe des filtres sur X contenant \mathcal S.
Il est clair d'après (F_2) qu'aucune intersection finie d'éléments de \mathcal S ne doit être vide.
Cette condition nécessaire est suffisante. Plus précisément :
Proposition 1 et définition
Pour qu'il existe un filtre sur X contenant un ensemble \mathcal S de parties de X, il faut et il suffit qu'aucune des intersections finies d'ensemble de \mathcal S ne soit vide.
Dans ces conditions, le plus petit (au sens de l'inclusion) filtre \mathcal F contenant \mathcal S est appelé le filtre engendré par \mathcal S et on dit que \mathcal S est un système générateur de ce filtre.
\mathcal F est l'intersection de tous les filtres qui contiennent \mathcal S.

Démonstration:
Notons \mathcal S' l'ensemble des intersections finies d'éléments de \mathcal S.
Notons \mathcal S'' l'ensemble des parties de X qui contiennent un élément de \mathcal S'.
Montrons que \mathcal S'' est le plus petit filtre contenant \mathcal S.

Tout d'abord, \mathcal S'' est un filtre. En effet :

Soit \Delta \in \mathcal S'' et P \supset \Delta.
Comme \Delta contient un élément \Delta' de \mathcal S' alors P \supset \Delta' et P\in \mathcal S'.
\mathcal S'' satisfait donc (F_1)

Il est évident que X\in \mathcal S''.
Soient alors A et B deux éléments de \mathcal S''.
A et B contiennent alors chacun respectivement un élément P et Q de \mathcal S'.
Ainsi, P \cap Q contient un élément R\in \mathcal S', et ce, par définition de \mathcal S'.
Il vient donc que P \cap Q \supset R et P\cap Q \in \mathcal S''
\mathcal S'' satisfait donc (F_2)

Comme par hypothèse, la partie vide de X n'est pas élément de \mathcal S', alors la partie vide de X n'est pas élément de \mathcal S''
\mathcal S'' satisfait donc (F_3).

Enfin, soit \mathcal F un filtre contenant \mathcal S.
Alors d'après (F_2), il contient \mathcal S'.
d'après (F_1), il contient toute partie de X qui contient un élément de \mathcal S'.
Autrement dit \mathcal F contient \mathcal S''.
Notations
Pour tout système générateur \mathcal S d'un filtre \mathcal F, on notera \mathcal F = \mathcal F(\mathcal S)
En particulier, dans un espace topologique (Y,\tau), pour tout y \in Y, on notera \mathcal V(y) le filtre des voisinages de y.
On a donc, si \mathcal B est un système fondamental de voisinage de y, \mathcal V(y)=\mathcal F(\mathcal B)

Exemple:
Soit \mathcal S un ensemble de parties quelconques d'un ensemble X et \mathcal T la topologie sur X engendrée par \mathcal S (Cette topologie existe toujours).
Comme l'ensemble des intersections finies d'ensemble de \mathcal S est une base de \mathcal T, il résulte que pour tout x\in X, \mathcal V(x) est engendré par l'ensemble \mathcal S(x) des ensembles de \mathcal S auxquels appartient x.
Corollaire 1
Soient \mathcal F un filtre sur un ensemble X et A une partie de X.
Les conditions suivantes sont équivalentes :

i) \text{ Il existe un filtre } \mathcal F'\text{ plus fin que } \mathcal F\text{ et qui contient } A
ii) A \text{ rencontre tous les éléments de } \mathcal F

Démonstration:
ii)\Rightarrow i)
On considère \mathcal S = \mathcal F \cup \lbrace A \rbrace et on vérifie aisément qu'aucune intersection finie d'élément de \mathcal S n'est vide.
Alors le filtre \mathcal F' engendré par \mathcal S convient.

i)\Rightarrow ii)
S'il existe un filtre \mathcal F' qui contient \mathcal F et dont A est élément,
cela signifie d'après (F_2) que A rencontre tous les éléments de \mathcal F' et donc de \mathcal F.

Exemple:
Dans \R muni de sa topologie usuelle, considérons \mathcal F le filtre des voisinages de 0 et \displaystyle A =\left\lbrace \dfrac{1}{n}  / n \in \N^* \right\rbrace
Il est clair que A rencontre chacun des éléments de \mathcal F.
Par conséquent il existe un filtre \mathcal F' plus fin que \mathcal F et qui contient A.
Corollaire 2
Soit \Phi une famille de filtres sur un ensemble non vide X.
Les conditions suivantes sont équivalentes :

i)\Phi \text{ admet une borne supérieure dans l'ensemble ordonné des filtres sur } X
ii) \text{ Pour toute suite finie } (\mathcal F_i)_{1\leq i \leq n}\text{ et tout } A_i \in \mathcal F_i \text{ on a } \displaystyle\bigcap_{i=1}^{i=n} A_i \neq \emptyset

Démonstration:
C'est immédiat car ce n'est que la traduction du fait que l'ensemble \displaystyle\bigcup_{i\in I} \mathcal F_i répond au critère de la proposition 1.

Exemple:
Pour tout x \in \R, notons \mathcal F_x = \lbrace \R,\R-\lbrace x \rbrace \rbrace.
L'ensemble \lbrace \mathcal F_x~|~x \in \R \rbrace est une famille de filtres sur \R.
Il est clair que pour tout famille finie (x_i)_{1\leq i \leq n} de réels et tout élément Y_i \in \mathcal F_{x_i}, \displaystyle\bigcap_{1\leq i \leq n} Y_i\neq \emptyset
Par suite, la famille (\mathcal F_x)_{x\in \mathbb{R}} admet une borne supérieure dans l'ensemble ordonné des filtres sur \R.
Cette borne supérieure n'est rien d'autre que le filtre des complémentaires des parties finies de \R.
Corollaire 3
L'ensemble ordonné des filtres sur un ensemble non vide X est inductif.

Démonstration:
En effet, considérons une famille (\mathcal F_i)_{i\in I } totalement ordonnée de filtres sur X.
Soient alors F_1,\cdots F_n des éléments respectifs de \mathcal F_{i_1},\cdots,\mathcal F_{i_n} où les i_p\in I sont différents deux à deux.
Comme la famille (\mathcal F_i)_{i\in I } est totalement ordonnée, alors il existe un indice i_0\in \lbrace i_1,\cdots i_n \rbrace tel que \forall i \in \N_n, F_i \in \mathcal F_{i_0}.
Alors l'intersection de la famille (F_i)_{1\leq i \leq n} est non vide et, en vertu du corollaire 2, la famille (\mathcal F_i)_{i\in I } admet une borne supérieure et donc un majorant dans l'ensemble ordonné des filtres sur X.

3. Base d'un filtre

Si \mathcal  S est un système générateur d'un filtre \mathcal F sur X, \mathcal F n'est en général pas l'ensemble des parties de X qui contiennent un élément de \mathcal S.

Par exemple, dans \R, considérons les deux parties A = [-1,1] et B = [0,2]
Comme A \cap B \neq \emptyset, alors il existe un filtre qui contient A et qui contient B.
Ce filtre contient en particulier l'ensemble C = [0,1] qui ne contient ni A ni B.
Définition
Un ensemble de parties \mathcal P sur un ensemble X est quasiment stable par intersection finie si l'intersection de deux éléments de \mathcal P contient un élément de \mathcal P.
On en déduit qu'un ensemble X est quasiment stable par intersection finie si et seulement si toute intersection finie d'éléments de \mathcal P contient un élément de \mathcal P

On voit donc que, dans l'exemple ci-dessus, l'ensemble \mathcal P =\lbrace A,B \rbrace n'est pas quasiment stable par intersection.
Considérons alors \displaystyle\mathcal P' =\left\lbrace A,B,\left[\dfrac{1}{2},\dfrac{3}{4}\right] \right\rbrace.
\mathcal P' est quasiment stable par intersection.
Proposition 2
Soit \mathcal B un ensemble de parties d'un ensemble X.
Pour que l'ensemble des parties de X contenant un ensemble de \mathcal B soit un filtre sur X, il faut et il suffit que \mathcal B satisfasse aux deux axiomes suivants :

\bullet(B_1) \mathcal B est quasiment stable par intersection finie.
\bullet (B_2) \mathcal B \neq \emptyset \text{ et } \emptyset \notin \mathcal B

Démonstration:
\mathcal B étant donné, considérons l'ensemble \mathcal F = \lbrace F \subset X~|~\exists B \in \mathcal B,~F\supset B \rbrace.

\boxed{\Rightarrow} Supposons que \mathcal F soit un filtre.
Alors, par définition même de \mathcal F, \mathcal B doit être non vide.
Si le vide était un élément de \mathcal B, \mathcal F serait égal à \mathcal P(X), ce qui est impossible.
Ainsi (B_2) est satisfait.

Montrons la quasi stabilité par intersection finie de \mathcal B.
Soient S et T deux éléments de \mathcal B.
Soit C = S \cap T.
Comme S,T\in \mathcal F et que \mathcal F est un filtre par hypothèse, alors C \in \mathcal F.
Donc par définition de \mathcal F, C contient un C' \in \mathcal B.
On a donc l'existence d'un C' \in \mathcal B tel que C' \subset S\cap T.
Comme S et T étaient arbitraires dans \mathcal B, on en déduit que \mathcal B est quasiment stable par intersection finie.
Ainsi (B_1) est satisfait.

\boxed{\Leftarrow} Supposons (B_1) \text{ et } (B_2)
Comme \mathcal B\neq \emptyset alors \mathcal F est non vide.
Comme \emptyset \notin \mathcal B alors tout élément de \mathcal F est non vide.
L'axiome (F_3) est donc satisfait.

Soit P \in \mathcal F et P' une partie de X contenant P.
Par définition de \mathcal F, il existe un B \in \mathcal B tel que P \supset B.
Il vient donc que P' \supset B et par suite P' \in \mathcal F.
L'axiome (F_1) est donc satisfait.

Soient maintenant A, B \in \mathcal F.
Il existe alors A', B' \in \mathcal B tels que A \supset A' et B \supset B'.
Par hypothèse \mathcal B est quasiment stable par intersection, donc il existe C' \in \mathcal B tel que A' \cap B' \supset C' et par suite A \cap B \supset A'\cap B' \supset C'.
L'axiome (F_{2a}) est donc satisfait.

Comme \mathcal B est non vide alors il existe B \in \mathcal B et comme B \subset X on conclut que X \in \mathcal F.
L'axiome (F_{2b}) est donc satisfait.

Conclusion: \mathcal F est un filtre.

Exemple:
Sur \R, on considère \displaystyle\mathcal I=(I_n)_{n\in \N^*}, la famile des intervalles de la forme \displaystyle I_n=\left]-\dfrac{1}{n},\dfrac{1}{n}\right[.
\mathcal I satisfait clairement les conditions de la proposition 1 et engendre donc un filtre sur \R.
On vérifie sans peine que \mathcal I est quasiment stable par intersection (et même stable).
Donc \mathcal F(\mathcal I) est l'ensemble des parties de \R qui contiennent un des I_n.
Ce filtre n'est rien d'autre que l'ensemble des voisinages de 0 pour \R muni de sa topologie canonique.
Définition 3
On dit qu'un ensemble de parties \mathcal B d'un ensemble X qui satisfait aux axiomes (B_1) et (B_2) est une base du filtre qu'il engendre.
On dit que deux bases de filtres sont équivalentes lorsqu'elles engendrent le même filtre.

Une base de filtre est donc un cas particulier de partie génératrice. C'est une partie génératrice quasiment stable par intersection.
Et si \mathcal S est une partie génératrice d'un filtre \mathcal F, alors l'ensemble \mathcal S' des intersections finies d'éléments de \mathcal S est une base de \mathcal F.
Proposition 3
Pour qu'une partie \mathcal B d'un filtre \mathcal F soit une base de ce filtre, il faut et il suffit que tout ensemble de \mathcal F contienne un ensemble de \mathcal B

Démonstration:
Supposons qu'une partie \mathcal B d'un filtre \mathcal F soit une base de ce filtre.
Soit F \in \mathcal F.
Alors par construction, F contient une intersection finie G d'éléments de \mathcal B.
Or par hypothèse, \mathcal B est une base et donc G contient un élément de \mathcal B.
Donc F contient un élément de \mathcal B.

Réciproquement, supposons que tout élément F \in \mathcal F contienne un élément de \mathcal B.
Remarquons d'abord que \mathcal B étant une partie de \mathcal F alors \emptyset \notin \mathcal B.
Remarquons ensuite que pour que tout ensemble de \mathcal F contienne un ensemble de \mathcal B, il est nécessaire que \mathcal B soit non vide.
Soient donc B_1,B_2 \in \mathcal B.
Alors, en particulier, B=B_1\cap B_2 \in \mathcal F et donc par hypothèse, B contient un élément \breve B \in \mathcal B
Proposition 4
Pour qu'un filtre \mathcal F' de base \mathcal B' soit plus fin qu'un filtre \mathcal F de base \mathcal B, il faut et il suffit que tout élémént de \mathcal B contienne un élément de \mathcal B'.

Démonstration:
Condition suffisante:
Tout élément de \mathcal F contenant un élément de \mathcal B, contient également un élément de \mathcal B', et c'est donc un élément de \mathcal F'.

Condition nécessaire:
Par hypothèse, tout élément de \mathcal F est élément de \mathcal F' et donc en particulier, tout élément de \mathcal B est élément de \mathcal F' et contient, en vertu de la proposition 3, un élément de \mathcal B'.
Corollaire
Pour que deux bases de filtre \mathcal B et \mathcal B' soient équivalentes, il faut et il suffit que tout élémént de \mathcal B contienne un élément de \mathcal B' et inversement.

4. Ultrafiltres

Définition 4
On appelle ultrafiltre sur X un filtre tel qu'il n'en n'existe pas de plus fin que lui.
Autrement dit, c'est un élément maximal de l'ensemble ordonné des filtres sur X.

Théorème 1
Pour tout filtre \mathcal F sur un ensemble X, il existe un ultrafiltre plus fin que \mathcal F

C'est une conséquence du fait que l'ensemble ordonné des filtres sur X est inductif et du théorème de Zorn.
Proposition 5
Soit \mathcal U un ultrafiltre sur un ensemble X.
Si A et B sont deux parties de X telles que A \cup B \in \mathcal U alors A \in \mathcal U ou B \in \mathcal U

Autrement dit, si U \in \mathcal U, alors pour tout recouvrement de U en deux sous-ensembles, l'un des deux éléments du recouvrement est un élément de \mathcal U.

Démonstration:
Raisonnons par l'absurde et supposons que A \notin \mathcal U, B \notin \mathcal U tandis que A\cup B \in \mathcal U.
Considérons l'ensemble \mathcal S des parties M de X telles que A \cup M \in \mathcal U.
On vérifie que \mathcal S est un filtre.
Il est clair que si U \in \mathcal U alors A \cup U \in \mathcal U et donc U \in \mathcal S et par suite \mathcal U\subset \mathcal S.
Or par hypothèse, B \in \mathcal S et donc \mathcal S est strictement plus fin que \mathcal U, ce qui est absurde.
Corollaire
Si la réunion d'une suite finie (A_i)_{1\leq i \leq n} de parties de X appartient à un ultrafiltre \mathcal U alors l'un au moins des A_i appartient à \mathcal U.
En particulier, si (A_i)_{1\leq i \leq n} est un recouvrement (une partition) de X, alors au moins l'un des A_i est un élément de \mathcal U puisque X \in \mathcal U

La proposition 5 caractérise les ultrafiltres. Et plus généralement on a le résultat suivant :
Proposition 6
Soit \mathcal S un système générateur d'un filtre sur un ensemble X.
On suppose que pour toute partie Y de X on a Y\in \mathcal S ou \complement Y\in \mathcal S.
Alors \mathcal S est un ultrafiltre sur X.

Démonstration:
En effet, soit \mathcal F un filtre contenant \mathcal S (il en existe par hypothèse).
Soit Y \in \mathcal F.
Dans ce cas \complement Y \notin \mathcal F et donc \complement Y \notin \mathcal S.
Ce qui entraîne par hypothèse que Y \in \mathcal S et donc \mathcal F \subset \mathcal S.
Comme on a trivialement \mathcal S \subset \mathcal F alors \mathcal S = \mathcal F.
Comme l'ajout d'une partie de X dans \mathcal S est impossible sous peine de tolérer la partie vide de X dans \mathcal S, on déduit que \mathcal S est un ultrafiltre.

Exemple d'ultrafiltre:
Soit X un ensemble non vide et a \in X.
L'ensemble \mathcal U_a des parties de X qui contiennent a est un ultrafiltre.
En effet, c'est un filtre et si Y \subset X alors a \in Y ou a \in \complement Y.
Par conséquent soit Y\in \mathcal U_a soit \complement Y \in \mathcal U_a.
Ces ultrafiltres sont appelés filtres triviaux.
Les ultrafiltres sur des ensembles finis sont tous triviaux.
Un ultrafiltre non trivial est appelé ultrafiltre libre.

Le théorème 1 nous dit que tout filtre (et plus généralement tout sous-ensemble ayant la propriété de quasi stabilité par intersection) est contenu dans un ultrafiltre, et par conséquent que des ultrafiltres libres existent, mais cette démonstration utilisant l'axiome du choix,il n'est pas possible de donner des exemples d'ultrafiltres non triviaux.
On peut toutefois noter qu'un ultrafiltre libre, sur un ensemble nécessairement infini X, ne contient pas de parties finies de X.
Par conséquent, en vertu du corollaire ci-dessus, un ultrafiltre est libre si et seulement s'il est plus fin que le filtre des compléméntaires des parties finies.

Si on admet l'axiome du choix, les propositions 5 et 6 montrent que «presque tous» les ultrafiltres sur un ensemble infini X sont libres;
plus précisément : le cardinal de l'ensemble des ultrafiltres (donc aussi celui des ultrafiltres libres) est égal à 2^{2^{|X|}} (strictement supérieur au cardinal |X| de l'ensemble des ultrafiltres triviaux).
En effet, un filtre \mathcal U sur X est un ultrafiltre si est seulement si pour toute partie Y \subset X, on a Y\in \mathcal U ou bien \complement Y\in \mathcal U.

A titre d'exemple, l'ensemble des ultrafiltres sur \N est équipotent à \mathcal P(\R).
C'est énorme, et pourtant on ne sait pas en construire qui soit non trivial autrement que par l'axiome du choix.

Par conséquent, dans la suite, on n'utilisera uniquement les propriétés des ultrafiltres telles qu'elles ont été décrites ci-dessus.
Proposition 7
Tout filtre \mathcal F est l'intersection des ultrafiltres plus fins que lui.

Démonstration:
Il est clair que cette intersection contient \mathcal F.
Soit alors A\notin \mathcal F et montrons que A n'est pas dans cette intersection.
Posons A' = \complement A.
Comme A ne contient aucun ensemble de \mathcal F, on a M \cap A'\neq\emptyset pour tout M\in \mathcal F et donc il existe un filtre \mathcal F' plus fin que \mathcal F et contenant A'.
Si maintenant \mathcal U est un ultrafiltre plus fin que \mathcal F', alors \complement A \in \mathcal U et donc A \notin \mathcal U
Par suite, on a construit un ultrafiltre plus fin que \mathcal F et ne contenant pas A.

5. Filtre induit

Proposition 8
Soient \mathcal F un filtre sur un ensemble X et A une partie de X.
Pour que la trace \mathcal F_A de \mathcal F sur A soit un filtre sur A, il faut et il suffit que tout ensemble de \mathcal F rencontre A.

Rappel : \mathcal F_A = \lbrace F\cap A~|~F \in \mathcal F \rbrace
En particulier, si A \in \mathcal F, alors \mathcal F_A est un filtre sur A.
Définition 5
Si la trace, sur une partie A d'un ensemble X, d'un filtre \mathcal F sur X, est un filtre sur A, on dit que ce filtre est induit par \mathcal F sur A.

Exemple important:
Soit (X,\tau) un espace topologique, A une partie de X et x \in X.
Pour que la trace sur A du filtre des voisinages du point x soit un filtre sur A, il faut et il suffit que tout voisinage de x rencontre A, autrement dit, que x \in \bar A.
Ce qui fait l'intérêt de cet exemple de filtre induit, c'est que, d'une part, il joue un rôle important dans la théorie des limites, et que d'autre part, tout filtre peut être défini de cette manière.
En effet, soit \mathcal F un filtre sur un ensemble X.
Soit X' l'ensemble obtenu en adjoignant à X un nouvel élément \omega.
Soit \mathcal F' le filtre sur X' formé des ensembles M \cup \lbrace \omega \rbrace M parcourt \mathcal F.
Pour tout point x \neq \omega de X', soit \mathcal V(x) l'ensemble des parties de X' contenant x.
Posons d'autre part \mathcal V(\omega) = \mathcal F'
Les \mathcal V(x) pour x parcourant X' définissent sur X' une topologie dont ils sont les filtres de voisinages.
Enfin, \omega est adhérent à X pour cette topologie et \mathcal F est induit par \mathcal F' = \mathcal V(\omega) sur X.
La topologie ainsi définie sur X' s'apelle topologie associée à \mathcal F.
Proposition 9
Pour qu'un ultrafiltre \mathcal U sur un ensemble X induise un filtre sur une partie A de X, il faut et il suffit que A \in \mathcal U.
Si cette condition est remplie, \mathcal U_A est un ultrafiltre sur A.

6. Images directe et réciproque d'une base de filtre

Proposition 10
Soit \mathcal B une base de filtre (resp. d'ultrafiltre) sur un ensemble X.
Soit f : X \rightarrow X' une application.
Alors f(\mathcal B) est une base de filtre (resp. d'ultrafiltre) sur X'

Démonstration:
Rappelons que f(\mathcal B) désigne l'ensemble \lbrace f(B)~|~B \in \mathcal B \rbrace.
Vérifions les axiomes (B_1) et (B_2) pour f(\mathcal B).
Comme \mathcal B est une base de \mathcal F alors la base n'est pas vide et ne contient pas l'ensemble vide. Donc tout B \in \mathcal B est non vide et par suite f(B) est non vide et f(\mathcal B) est non vide et ne contient pas le vide.
Par ailleurs, si A,B \in \mathcal B,\text{ alors }f(A),f(B)\in f(\mathcal B) et on a l'existence d'un C \in \mathcal B tel que C \subset A \cap B.
La relation f(A\cap B)\subset f(A) \cap f(B) montre que l'intersection de deux éléments de f(\mathcal B) contient un élément de f(\mathcal B).

Vérifions ce qu'il se passe si \mathcal B est une base d'ultrafiltre.
Considérons M' une partie de X'.
Si f^{-1}(M') contient un ensemble M \in \mathcal B, alors M' contient f(M)
Sinon \complement f^{-1}(M') = f^{-1}(\complement M') contient un ensemble N \in \mathcal B (cf proposition 5) et donc \complement M' contient f(N).
Ainsi, M'\in f(\mathcal B) ou \complement M'\in f(\mathcal B) et le résultat est alors une conséquence de la proposition 6.

Voyons ce qu'il en est des images réciproques de base de filtre.
Proposition 10b
Soit \mathcal B' une base de filtre sur un ensemble X'
Soit f : X \rightarrow X' une application.
Pour que f^{-1}(\mathcal B') soit une base de filtre sur X, il faut et il suffit que \forall B'\in \mathcal B',f^{-1}(B')\neq \emptyset

Cela résulte de la relation f^{-1}(M'\cap N') = f^{-1}(M') \cap f^{-1}(N').
Cette condition peut s'exprimer en disant que tout ensemble de \mathcal B' rencontre f(X).
Ou encore que la trace de \mathcal B' sur f(X) est une base de filtre.

7. Produit de filtres.

Définition 6
Soit (X_i)_{i\in I} une famille d'ensemble et pour tout i \in I, \mathcal F_i un filtre sur X_i.
On appelle produit des filtres \mathcal F_i et l'on note \displaystyle\prod_{i\in I}\mathcal F_i (si aucune confusion n'en résulte) le filtre sur \displaystyle X = \prod_{i\in I} X_i ayant pour base l'ensemble des parties de la forme \displaystyle\prod_{i\in I} M_i, où M_i \in \mathcal F_i pour tout i \in I et M_i = X_i sauf pour un nombre fini d'indices.

Cette définition a un sens en vertu de la formule \displaystyle\left(\prod_{i\in I} M_i \right)\cap \left(\prod_{i\in I} N_i \right)= \prod_{i\in I} (M_i \cap N_i)

8. Filtres élémentaires.

Définition 7
Soit (x_n)_{n\in \N} une suite infinie d'éléments d'un ensemble X.
On appelle filtre élémentaire associé à la suite (x_n)_{n\in \N} le filtre engendré par l'image du filtre de Fréchet par l'application n \mapsto x_n de \N dans X.

Autrement dit, le filtre élémentaire associé à (x_n)_{n\in \N} est l'ensemble des parties M de X telles que l'on ait x_n\in M sauf pour un nombre fini de valeurs de n
Si S_n= \lbrace x_p~|~p \geq n \rbrace, les ensembles S_n forment une base du filtre élémentaire associé à (x_n)_{n\in \N}.
Tout filtre élémentaire possède donc, par définition, une base dénombrable. Inversement :
Proposition 11
Si un filtre \mathcal F possède une base dénombrable, il est le filtre intersection des filtres élémentaires plus fin que \mathcal F

§2 - Les Limites.

1. Limite d'un filtre

Définition 1
Soient X un espace topologique, \mathcal F un filtre sur X et x \in X.
On dit que x est point limite (ou limite) de \mathcal F, si \mathcal F est plus fin que le filtre \mathcal V(x) des voisinages de x. On dit alors que \mathcal F converge (ou est convergent) vers x.
On dit que x est limite d'une base de filtre \mathcal B sur X (ou que \mathcal B converge vers x) si le filtre \mathcal F(\mathcal B) converge vers x.

Très intuitivement, on peut dire qu'un filtre converge vers x s'il contient des ensembles qui sont de plus en plus voisins autour de x.

Il résulte immédiatement de la proposition 4 du paragraphe précédent ceci :
Proposition 1
Pour qu'une base de filtre \mathcal B sur un espace topologique X converge vers x \in X, il faut et il suffit que tout ensemble d'un système fondamental de voisinage de x contienne un ensemble de \mathcal B

Si un filtre \mathcal F sur X converge vers x, alors il en est de même de tout filtre plus fin que \mathcal F.

De même, si on remplace la topologie de X par une topologie moins fine, le filtre des voisianges de x est remplacé par un filtre moins fin, donc \mathcal F converge encore vers x pour cette topologie.

Si \Phi est une famille de filtres sur X qui convergent tous vers un même point x, alors le filtre \mathcal V(x) est moins fin que les filtres de \Phi et est donc aussi moins fin que le filtre \mathcal I intersection des filtres de la famille \Phi.
Autrement dit, \mathcal I converge aussi vers x.

Avec la proposition 7 du paragraphe précédent, il résulte ceci :
Proposition 2
Pour qu'un filtre \mathcal F sur un espace topologique X converge vers un point x \in X, il faut et il suffit que tout ultrafiltre plus fin que \mathcal F converge vers x

2. Point adhérent à une base de filtre

Définition 2
Soit X un espace topologique, \mathcal P un ensemble de parties de X et x\in X.
On dit que x est adhérent à \mathcal P s'il est adhérent à tous les ensembles de \mathcal P.

En particulier, si x est adhérent à une base de filtre \mathcal B sur X, alors il est adhérent au filtre de base \mathcal B et donc à toute base de filtre équivalente.
Il résulte des définitions que :
Proposition 3
Pour qu'un point x soit adhérent à une base de filtre \mathcal B, il faut et il suffit que tout ensemble d'un système fondamental de voisinage de x rencontre chacun des ensembles de \mathcal B

Il en résulte immédiatement, avec le corollaire 1 du paragraphe précédent que :
Proposition 4
Pour qu'un point x soit adhérent à un filtre \mathcal F, il faut et il suffit qu'il existe un filtre plus fin que \mathcal F et qui converge vers x.

Démonstration:
Supposons que x soit adhérent à un filtre \mathcal F.
Désignons par \mathcal V(x) le filtre des voisinages de x.
Alors \mathcal V(x) \cup \mathcal F est une base de filtre sur X en vertu de la proposition 3.
Le filtre engéndré par cette base est clairement plus fin que \mathcal F et converge vers x.

Réciproquement, supposons qu'il existe un filtre \mathcal F' plus fin que \mathcal F et qui converge vers x.
Alors les éléments de \mathcal F sont des éléments de \mathcal F', lequel contient \mathcal V(x)
Donc si x \in X et F \in \mathcal F, F rencontre tout voisinage de x dans \mathcal F', et par conséquent x est adhérent à F

En particulier, tout point limite d'un filtre \mathcal F est un point adhérent à \mathcal F.
Corollaire
Pour qu'un point x soit adhérent à un ultrafiltre \mathcal U, il faut et il suffit que x soit point limite de \mathcal U.

Démonstration:
En effet, si x est adhérent à \mathcal U, alors il existe un filtre \mathcal U' plus fin que \mathcal U et qui converge vers x.
Comme \mathcal U est un ultrafiltre, alors \mathcal U' = \mathcal U et \mathcal U converge vers x.

L'ensemble des points adhérents à une base de filtre \mathcal B est par définition l'ensemble \displaystyle\bigcap_{B \in \mathcal B}\overline B, d'où :
Proposition 5
L'ensemble des points adhérents à une base de filtre sur un espace topologique X est fermé dans X.

Proposition 6
Soit \mathcal B une base de filtre sur une partie A d'un espace topologique X.
Alors tout point adhérent à \mathcal B dans X appartient à \overline A.
Inversement, tout point de \overline A est limite dans X d'un filtre sur A.

Démonstration:
La première assertion est triviale.
D'autre part, si x \in \bar A, la trace sur A du filtre des voisinages de x dans X est un filtre sur A qui converge évidemment vers x.

3. Valeur limite et valeur d'adhérence d'une fonction

Définition 3
Soient (X,\mathcal F) un ensemble filtré et (Y,T) un espace topologique.
Soit f : X \rightarrow Y une application et y \in Y.

On dit que y est valeur limite (ou simplement limite) de f suivant le filtre \mathcal F si la base de filtre f(\mathcal F) converge vers y.
Autrement dit, dans Y,~y est limite du filtre \mathcal F(f(\mathcal F))

On dit que y est valeur d'adhérence de f suivant le filtre \mathcal F si y est un point adhérent à la base de filtre f(\mathcal F).

La relation
y \text{ est limite de }f \text{ suivant le filtre }\mathcal F

S'écrit
~~\underset{\mathcal F}\lim~ f = y

Ou bien
~~\underset{x,\mathcal F}\lim~ f(x) = y

Ou encore (si aucune confusion n'en résulte)
~~\underset{x}\lim~ f(x) = y


Voyons tout de suite un exemple simple d'utilisation de la définition.

Exemple important:
Considérons la fonction définie sur \R par f(x) = 1 si x = 0, f(x) = 0 sinon.
L'espace d'arrivée est muni de la topologie usuelle.
Prenons trois filtres différents sur \R et voyons quelle est la limite de f suivant ces trois filtres.

1er filtre: le filtre des voisinages de 0.
Notons que ce filtre converge vers 0.
L'image de ce filtre par f est l'ensemble \lbrace 0,1 \rbrace qui est la base d'un filtre sur \R qui ne converge pas.
En effet, il est assez aisé de vérifier qu'il n'est pas plus fin qu'aucun filtre de voisinage de point.
Selon ce filtre, f n'a pas de limite et possède deux valeurs d'adhérence que sont 0 et 1.

2d filtre: le filtre dont une base est donnée par l'ensemble des voisinages épointés de 0.
Notons que ce filtre est plus fin que le filtre des voisinages de 0 et converge donc vers 0.
L'image de ce filtre par f est l'ensemble \lbrace \lbrace 0 \rbrace,\lbrace 0,1 \rbrace \rbrace qui est une base de l'ultrafiltre trivial de base \lbrace 0 \rbrace, lequel est plus fin que le filtre des voisinages de 0.
Selon ce filtre, f admet 0 pour valeur limite et pour valeur d'adhérence.

3ème filtre: le filtre trivial de base \lbrace 0 \rbrace.
Notons que ce filtre est plus fin que le filtre des voisinages de 0 et converge donc vers 0.
L'image de ce filtre par f est l'ensemble \lbrace \lbrace 1 \rbrace,\lbrace 0,1 \rbrace \rbrace qui est une base de l'ultrafiltre trivial de base \lbrace 1 \rbrace, lequel est plus fin que le filtre des voisinages de 1.
Selon ce filtre, f admet 1 pour valeur limite et pour valeur d'adhérence.

Proposition 7

\bullet Pour que y \in Y soit limite de f suivant le filtre \mathcal F, il faut et il suffit que pour tout voisinage V de y dans Y, il existe un ensemble M \in \mathcal F tel que f(M) \subset V
(ou encore pour tout voisinage V de y dans Y,~f^{-1}(V) \in \mathcal F).
\bullet Pour que y \in Y soit valeur d'adhérence de f suivant \mathcal F, il faut et il suffit que pour tout voisinage V de y dans Y et tout ensemble M \in \mathcal F, il existe x\in M tel que f(x) \in V.


Démonstration:
Par définition, y \in Y est limite de f suivant le filtre \mathcal F signifie que \mathcal V(y) \subset \mathcal F(f(\mathcal F)) dans Y.
Si donc V \in \mathcal V(y) alors il existe un M \in \mathcal F tel que f(M) \subset V.
Ceci équivaut encore à M \subset f^{-1}(V) et donc que f^{-1}(V) \in \mathcal F par \bf{(F_1)}.

Exemple 1:
Une suite de points (x_n)_{n\in \N} d'un espace topologique X est une application n \mapsto x_n de \N dans X.
On a souvent à considérer, en Analyse, la notion de valeur limite ou de valeur d'adhérence d'une telle application suivant le filtre de Fréchet sur \N.
Si x est limite de l'application n\mapsto x_n suivant le filtre de Fréchet, on dit que x est limite de la suite (x_n) lorsque n croît indéfiniment (ou que (x_n) tend vers x lorsque n croît indéfiniment) et l'on écrit \displaystyle\underset{n \rightarrow \infty}\lim x_n = y.

On appelle de même valeur d'adhérence de la suite (x_n)_n toute valeur d'adhérence de l'applicaiton n\mapsto x_n suivant le filtre de Fréchet.

Si y est limite (resp. valeur d'adhérence) d'une application f : X \rightarrow Y, suivant un filtre \mathcal F sur X, alors y reste valeur limite (resp. valeur d'adhérence) de f suivant \mathcal F si on remplace la topologie de Y par une topologie moins fine.

De même, si y est limite (resp. valeur d'adhérence) d'une application f : X \rightarrow Y, suivant un filtre \mathcal F sur X, alors y reste valeur limite (resp. valeur d'adhérence) de f suivant tout filtre plus fin (resp. moins fin) que \mathcal F.


Exemple 2:
Considérons la fonction définie sur \R par f(x) = \sin(1/x) si x \neq 0 et f(0) = 10.
Considérons \mathcal V(0) le filtre des voisinages de 0.
On a f(\mathcal V(0)) = [-1,1]\cup \lbrace 10 \rbrace.
Par conséquent, l'ensemble des valeurs d'adhérence de f suivant \mathcal V(0) est l'ensemble [-1,1] \cup \lbrace 10 \rbrace.
en revanche, suivant ce filtre, f n'a pas de valeur limite.

Considérons \mathcal F_0 le filtre trivial de base \lbrace 0 \rbrace.
L'image f(\mathcal F_0) est le filtre trivial de base \lbrace 10 \rbrace sur [-1,1]\cup \lbrace 10 \rbrace, lequel est une base du filtre trivial de base \lbrace 10 \rbrace sur \R, plus fin que le filtre des voisinages de 10.
Par conséquent, suivant \mathcal F_0,~f a une et une seule valeurs d'adhérence qui est 10, et qui est aussi valeur limite.

Remarques:
\bullet si y est valeur limite d'une application f : X \rightarrow Y suivant le filtre \mathcal F \text{ sur }X, alors y reste valeur limite de f suivant \mathcal F quand on remplace la topologie de Y par une topologie moins fine.

\bullet si y est valeur limite d'une application f : X \rightarrow Y suivant le filtre \mathcal F \text{ sur }X, alors y reste valeur limite de f suivant tout filtre plus fin que \mathcal F.

\bullet si y est valeur d'adhérence d'une application f : X \rightarrow Y suivant le filtre \mathcal F \text{ sur }X, alors y reste valeur d'adhérence de f suivant \mathcal F quand on remplace la topologie de Y par une topologie moins fine.

\bullet si y est valeur d'adhérence d'une application f : X \rightarrow Y suivant le filtre \mathcal F \text{ sur }X, alors y reste valeur d'adhérence de f suivant tout filtre moins fin que \mathcal F.

Proposition 8
Soient (X,\mathcal F) un ensemble filtré et (Y,T) un espace topologique.
Soit f : X \rightarrow Y une application et y \in Y.
Pour que y soit valeur d'adhérence de f suivant \mathcal F, il faut et il suffit qu'il existe sur X un filtre \mathcal S plus fin que \mathcal F et tel que y soit limite de f suivant \mathcal S

Notons enfin que l'ensemble des valeurs d'adhérence de f suivant un filtre, est fermé dans Y.

4. Limite et continuité.

Pour ce paragraphe on se place dans le cas particulierf est une application entre deux espaces topologiques, et \mathcal V(a) le filtre des voisinages d'un point a \in X.

Au lieu de dire que " \ell est limite de f suivant \mathcal V(a) " et d'écrire \underset{\mathcal V(a)}\lim f = \ell, on utilise la notation particulière :

\boxed {\ell = \underset{x\rightarrow a}\lim~ f(x)}
qui correspond à la définition suivante :

\boxed {\forall V '\in \mathcal V(\ell),~\exists V \in \mathcal V(a),~(\forall x)(x\in V \Rightarrow f(x) \in V')}

et on dit que
\ell\text{ est limite de }f \text{ au point }a

ou que
f(x)\text{ tend vers } \ell \text{ quand }x \text{ tend vers }a.


De même, au lieu de dire que y est valeur d'adhérence de f suivant \mathcal V(a), on dit que " y est valeur d'adhérence de f au point a ".

Par suite f(a) est toujours valeur d'adhérence de f au point a. Plus précisément :
Proposition 9
Soit f:X\rightarrow Y une application entre deux espaces topologiques et a \in X.
Les propositions suivantes sont équivalentes :

i)~ f\text{ est continue au point }a
{ii)~\underset{x\rightarrow a}\lim f(x) = f(a).

Démonstration:
En effet, dans la proposition 7, prenons y = f(a).
Par définition de la continuité, f sera continue en a si et seulement si pour tout voisinage V' de f(a) dans Y, il existe un voisinage V de a dans X tel que f^{-1}(V') = V.
La proposition 7 nous dit que c'est encore équivalent à ce que f admette une limite pour le filtre des voisinages de a dans X et que cette limite est f(a).
corollaire 1
Soient X,Y deux espaces topologiques, f:X\rightarrow Y et a \in X.
On suppose que f est continue au point a.
Alors pour toute base de filtre \mathcal B sur X qui converge vers a, la base de filtre f(\mathcal B) converge vers f(a).
Inversement, si, pour tout ultrafiltre \mathcal U sur X qui converge vers a, la base de filtre f(\mathcal U) converge vers f(a), alors f est continue en a.

Démonstration:
La première assertion est une conséquence immédiate de la proposition 9.

Pour démontrer la seconde, supposons que f ne soit pas continue au point a.
Il existe alors un voisinage W de f(a) dans Y tel que f^{-1}(W) ne soit pas un élément du filtre \mathcal V(a) des voisianges de a dans X.
On a vu qu'il existait un ultrafiltre \mathcal U plus fin que \mathcal V(a) et ne contenant pas f^{-1}(W), donc, contenant son complémentaire A = X - f^{-1}(W).
Comme f(A)\cap W = \emptyset,~f(\mathcal U) alors W \notin f(\mathcal U) et f(\mathcal U) ne converge pas vers f(a).
corollaire 2
Soit X un espace topologique, Y,Z deux ensembles et a \in X.
Soit \mathcal F un filtre sur Z.
Soit g : Z \rightarrow X admettant a comme limite suivant le filtre \mathcal F.
Soit f : X \rightarrow Y une application continue en a.

Alors f \circ g admet f(a) comme limite suivant le filtre \mathcal F.

Démonstration:
Par hypothèse g(\mathcal F) converge vers a dans X.
Comme f est continue en a, le corollaire 1 nous dit que f(g(\mathcal F)) converge vers f(a) dans Y.

5. Limite relativement à un sous-espace.

On considère ici deux espaces topologiques X et Y, A une partie de X, et a un point de X tel que a \in \bar A (mais n'appartenant pas nécessairement à A).
Soit \mathcal V_A(a) la trace sur A du filtre des voisinages de a dans X.
Soit f : A \rightarrow Y une application et \ell \in Y.

Au lieu de dire que " \ell est limite de f suivant \mathcal V_A(a) " et d'écrire \underset{\mathcal V_A(a)}\lim f = \ell, on écrit :

\boxed{\ell = \underset{\underset{x\in A}{x\rightarrow a}}\lim~f(x)}
qui correspond à la définition suivante :

\boxed {\forall V '\in \mathcal V(\ell),~\exists V \in \mathcal V(a),~(\forall x)(x\in V\cap A \Rightarrow f(x) \in V')}

et on dit que
 \ell\text{ est limite de }f \text{ au point }a, \text{ relativement au sous-espace }A

ou que
 f(x)\text{ tend vers }\ell \text{ lorsque }x \text{ tend vers }a, \text{ en restant dans }A


On remarquera que l'on a alors \ell\in \overline{f(A)}.
Lorsque A = \complement \lbrace a \rbrace, où a est un point non isolé de X, au lieu d'écrire y = \underset{\underset{x\in A}{x\rightarrow a}}\lim~f(x), on écrit aussi :

 \boxed{y = \underset{\underset{x\neq a}{x\rightarrow a}}\lim~f(x)}
qui correspond à la définition suivante :

\boxed {\forall V '\in \mathcal V(\ell),~\exists V \in \mathcal V(a),~(\forall x)(x\in V-\lbrace a \rbrace \Rightarrow f(x) \in V')}

Cette limite prend parfois le nom de limite de f en a selon les voisinages épointés de a.

Mais attention au côté abusif de cette dénomination; en effet l'ensemble des voisinages épointés de a n'est pas un filtre, mais la base d'un filtre strictement plus fin que celui des voisinages de a sous réserve d'existence tel que le stipule la proposition 1 du §1.
un tel filtre existera si et seulement si a n'est pas un point isolé de X. On peut alors l'appeler le filtre des voisinages épointés de a.

Soit a un point non isolé de X.
Si une fonction f admet une limite selon les voisinages de a, alors f admet la même limite selon le filtre des voisinages épointés de a (remarque 2 ci-dessus).L'inverse est inexact.
Plus précisément :
Proposition 9bis
Soit f:X\rightarrow Y une application entre deux espaces topologiques et a \in X un point non isolé.
Les propositions suivantes sont équivalentes :

i)~f\text{ est continue au point }a
ii)~\underset{x\rightarrow a,x\neq a}\lim f(x) = f(a).

Démonstration:
Notons \mathcal F_a = \mathcal F(\lbrace V-\lbrace a \rbrace~|~V\in \mathcal V(a)\rbrace).
Comme a est non isolé, \mathcal F_a est un filtre sur X-\lbrace a \rbrace.

i)\Rightarrow ii)
Si V est un voisinage de f(a) alors f^{-1}(V) est un voisinage de a et donc f^{-1}(V)-\lbrace a \rbrace \in \mathcal F_a.
Or on a f(V) \supset f(f^{-1}(V)-\lbrace a \rbrace)
Comme V est arbitraire, cela signifie que f(\mathcal F_a) est une base d'un filtre plus fin que les voisinages de f(a) dans Y et donc que \underset{x\rightarrow a,x\neq a}\lim f(x) = f(a).

ii)\Rightarrow i).
Posons y = f(a) et V' un voisinage arbitraire de y dans Y
On pose \tilde f : X-\lbrace a \rbrace \rightarrow Y,~\tilde f(x) = f(x)
Par la proposition 7, il vient que \tilde f^{-1}(V') \in \mathcal F_a.
Il existe donc V \in \mathcal V(a) tel que \tilde f^{-1}(V') \supset V-\lbrace a \rbrace.
On conclut que comme f^{-1}(V')\supset \tilde f^{-1}(V')\cup \lbrace a \rbrace alors f^{-1}(V') est un voisinage de a dans X.

Une autre façon de démontrer la proposition 9bis eut été de se contenter de montrer ii)\Rightarrow i).
En effet, dans ce cas et compte tenu de la proposition 9, on a imméditement les équivalences :

\boxed {f\text{ est continue au point }a\Leftrightarrow \underset{x\rightarrow a,x\neq a}\lim f(x) = f(a) \Leftrightarrow \underset{x\rightarrow a}\lim f(x) = f(a)}
Corollaire
Si une fonction admet une limite selon les voisinages épointés de a, mais pas selon les voisinages de a, alors f est discontinue en a.

Exemples:
On considère une application f : \R \rightarrow \R.

1)
Si la quantité \underset{\underset{x\in [a,+\infty[}{x\rightarrow a}}\lim~f(x)} existe, on dira que f admet une limite à droite en  a et on la notera :
\boxed{\underset{\underset{a\leq x}{x\rightarrow a}}\lim~f(x)}}
Cette limite correspond à la définition classique suivante :

\forall \varepsilon > 0, \exists \eta >0, \forall x \in \R,0 \leq x-a < \eta \Rightarrow |\ell - f(x) | < \varepsilon
Dans ce cas cette limite est égale à f(a) et f est continue à droite.


2)
Si la quantité \underset{\underset{x\in ]a,+\infty[}{x\rightarrow a}}\lim~f(x)} existe, on dira que f admet une limite épointée à droite en a et on la notera :
\boxed{\underset{\underset{a < x}{x\rightarrow a}}\lim~f(x)\text{ ou }\underset{x\rightarrow a^+}\lim~f(x)\text{ ou } f(a+)}
Cette limite correspond à la définition suivante :

\forall \varepsilon > 0, \exists \eta >0, \forall x \in \R,0 < x-a < \eta \Rightarrow |\ell - f(x) | < \varepsilon
Dans ce cas cette limite n'est pas nécessairement égale à f(a).


3)
Si la quantité \underset{\underset{x\in ]-\infty,a]}{x\rightarrow a}}\lim~f(x)} existe, on dira que f admet une limite à gauche en a et on la notera :
\boxed{\underset{\underset{x \leq a}{x\rightarrow a}}\lim~f(x)}}
Cette limite correspond à la définition classique suivante :

 \forall \varepsilon > 0, \exists \eta >0, \forall x \in \R,0 \leq a-x < \eta \Rightarrow |\ell - f(x) | < \varepsilon
Dans ce cas cette limite est égale à f(a) et f est continue à gauche.


4)
Si la quantité \underset{\underset{x\in ]-\infty,a[}{x\rightarrow a}}\lim~f(x)} existe, on dira que f admet une limite épointée à gauche en a et on la notera :
\boxed{\underset{\underset{x<a}{x\rightarrow a}}\lim~f(x) \text{ ou } \underset{x\rightarrow a^-}\lim~f(x)\text{ ou } {f(a-)}}
Cette limite correspond à la définition classique suivante :

 \forall \varepsilon > 0, \exists \eta >0, \forall x \in \R,0 < a-x < \eta \Rightarrow |\ell - f(x) | < \varepsilon
Dans ce cas cette limite n'est pas nécessairement égale à f(a).
Remarque-Corollaire
Soit f : \R \rightarrow \R une application et a \in \R.
f admet une limite selon les voisinages épointés de a si et seulement si f(a+)=f(a-).
f admet une limite selon les voisinages de a si et seulement si f(a+)=f(a-)=f(a) si et seulement si f est continue au point a.

Publié le
ceci n'est qu'un extrait
Pour visualiser la totalité des cours vous devez vous inscrire / connecter (GRATUIT)
Inscription Gratuite se connecter
Merci à
jsvdb
pour avoir contribué à l'élaboration de cette fiche


Vous devez être membre accéder à ce service...

Pas encore inscrit ?

1 compte par personne, multi-compte interdit !

Ou identifiez-vous :


Rester sur la page

Inscription gratuite

Fiches en rapport

parmi 1675 fiches de maths

Désolé, votre version d'Internet Explorer est plus que périmée ! Merci de le mettre à jour ou de télécharger Firefox ou Google Chrome pour utiliser le site. Votre ordinateur vous remerciera !